This problem deals with functions defined by f(x) = x^3 - 3bx with b > 0.

  • Thread starter Thread starter IntegrateMe
  • Start date Start date
  • Tags Tags
    Functions
Click For Summary
The discussion focuses on the function f(x) = x^3 - 3bx, where b > 0, and involves finding the coordinates of relative maximum and minimum points. The relative maximum occurs at x = -sqrt(b) with coordinates (-sqrt(b), 2b^(3/2)), while the relative minimum occurs at x = sqrt(b) with coordinates (sqrt(b), -2b^(3/2)). For part (c), participants clarify that the relative maximum and minimum points can be expressed in the form y = -2x^3 by substituting x values from parts (a) and (b). The conversation emphasizes the importance of correctly interpreting maximum and minimum values in relation to the function rather than just their coordinates. Overall, the thread effectively addresses the mathematical problem while seeking clarification on the final part.
IntegrateMe
Messages
214
Reaction score
1
(a) Find the x- and y-coordinates of the relative maximum points of f in terms of b.
(b) Find the x- and y-coordinates of the relative minimum points of f in terms of b.
(c) Show that for all values of b > 0, the relative maximum and minimum points lie on a function of the form y = -ax3 by finding the value of a.

(a)

f(x)=x3-3bx
f'(x)=3x2-3b=0
x2=b
x=+/-sqrt(b)

when x=-sqrt(b),
f(x) = y = -b3/2 - 3b(-b1/2)
(x,y) = ( -sqrt(b) , 2b3/2) f has a maximum

(b)

when x=sqrt(b),
f(x) = y = b3/2 - 3b(b1/2)
(x,y)=(sqrt(b),-2b3/2) f has a minimum

(c)

I'm not sure. Can someone help me with (c)?

Ok, i attempted each part. Did i do anything wrong?
 
Physics news on Phys.org
(a) and (b) are right. For (c), just use x=sqrt(b) from part (b) and express the y-value in terms of x: y = -2b3/2 = -2x3. Repeat the same with (a) to cover negative x values, the result will be the same.
 
Your terminology is confusing. When we speak of a maximum of ##f##, we don't mean a point on the plane, but the value of the function. So, the maximum of ##f## is ##f(x_0)## at the point ##x_0##.

For c) assume ##b>0## and repeat what you did before.
 
Question: A clock's minute hand has length 4 and its hour hand has length 3. What is the distance between the tips at the moment when it is increasing most rapidly?(Putnam Exam Question) Answer: Making assumption that both the hands moves at constant angular velocities, the answer is ## \sqrt{7} .## But don't you think this assumption is somewhat doubtful and wrong?

Similar threads

  • · Replies 3 ·
Replies
3
Views
956
  • · Replies 4 ·
Replies
4
Views
2K
Replies
8
Views
2K
  • · Replies 8 ·
Replies
8
Views
2K
Replies
2
Views
2K
  • · Replies 5 ·
Replies
5
Views
2K
  • · Replies 10 ·
Replies
10
Views
2K
Replies
2
Views
1K
  • · Replies 6 ·
Replies
6
Views
2K
  • · Replies 12 ·
Replies
12
Views
2K